Parallelogram ABCD is below. m

Parallelogram ABCD Is Below. M

Answers

Answer 1

The consecutive angles of a parallelogram are supplementary. therefore:

[tex]\begin{gathered} m\angle A+m\angle B=180 \\ 41+x+8.5=180 \\ x+49.5=180 \\ solve_{\text{ }}for_{\text{ }}x\colon \\ x=180-49.5 \\ x=130.5 \\ x\approx131 \end{gathered}[/tex]


Related Questions

Bryce is cutting tree trunks into circular pieces of wood 1 inch thick to make wall art for log cabins. Match the circumferences of each wood circle to its diameter or radius.

Answers

The circumference of a circle of radius r can be calculated as:

[tex]C=2\pi r[/tex]

If the diameter d is given, then the formula is:

[tex]C=\pi d[/tex]

Calculate the circumference of the following circles:

1 d = 8 inches.

[tex]\begin{gathered} C=\pi(8\text{ inches}) \\ C=3.14\cdot8\text{ inches} \\ C=25.12\text{ inches} \end{gathered}[/tex]

1 matches with b

2 d = 7 inches

[tex]\begin{gathered} C=\pi(7\text{ inches}) \\ C=3.14\cdot7\text{ inches} \\ C=21.98\text{ inches} \end{gathered}[/tex]

2 matches with d

3 r = 2 inches

[tex]\begin{gathered} C=2\pi(2\text{ inches\rparen=4}\cdot3.14\text{ inches} \\ C=12.56\text{ inches} \end{gathered}[/tex]

3 matches with a

4 r = 3 inches

[tex]\begin{gathered} C=2\pi(3\text{ inches\rparen=6}\cdot3.14\text{ inches} \\ C=18.84\text{ inches} \end{gathered}[/tex]

4 matches with c

Write an exponential equation using y = a(b)^x“ thatrepresents the growth or decay of the situation.A house was purchased for $370,000. The house has anannual appreciation rate of 3%. Please write a let statementand an equation that represents the house's value over time.

Answers

Let:

PV = Initial value = 370000

r = appreciation rate = 3% = 0.03

x = time

the equation will be given by

[tex]\begin{gathered} y=PV(1+r)^x \\ so\colon \\ y=370000(1+0.03)^x \\ y=370000(1.03)^x \end{gathered}[/tex]

Determine the domain and range of the quadratic function. f(x)=−2(x+8)^2−4

Answers

[tex]\begin{gathered} \text{Given} \\ f(x)=-2(x+8)^2-4 \end{gathered}[/tex]

Since the given is a polynomial with a degree of 2, there are no restrictions to its domain. The domain therefore is

[tex]\text{Domain: }(-\infty,\infty)[/tex]

The given function is in the vertex form

[tex]\begin{gathered} f(x)=a(x-h)^2+k \\ \text{where} \\ (h,k)\text{ is the vertex} \end{gathered}[/tex]

By inspection, we determine that the vertex of the function is at (-8,-4), and since a = -2, then the parabola opens up downwards. This implied that its output peaks at y = -4, and the graph continues towards negative infinity.

We can conclude therefore that the range is

[tex]\text{Range: }(-\infty,-4\rbrack[/tex]

For the polynomial below, -3 and 1 are zeros. Express f (x) as a product of linear factors.

Answers

Explanation

Since -3 and 1 are zeros of the functions, it implies that

[tex](x+3)\text{ }and\text{ }(x-1)[/tex]

are factors of the equation.

Therefore we can find the remaining factors below

[tex](x+3)(x-1)=x^2+2x-3[/tex]

By long division

[tex]remaining\text{ expression =}\frac{x^4+6x^3+7x^2-8x-6}{x^2+2x-3}=x^2+4x+2[/tex]

By quadratic formula

[tex]\begin{gathered} x_{1,2}=\frac{-4\pm\sqrt{4^2-4\times1\times2}}{2\times1} \\ x_1=\frac{-4+2\sqrt{2}}{2},x_2=\frac{-4-2\sqrt{2}}{2} \\ x=-2+\sqrt{2},x=-2-\sqrt{2} \\ therefore \\ (x+2-\sqrt{2})(x+2+\sqrt{2}) \end{gathered}[/tex]

The linear factor are

Answer:

[tex]f(x)=(x+3)(x-1)(x+2-\sqrt{2})(x+2+\sqrt{2})[/tex]

Please help I was sick and missed out on class.Thank you

Answers

The slope of the line passing through given points is 5/7

We know that the slope is the ratio of the change in y-values to the change in x-values.

We use slope formula,

m = (y2 - y1)/(x2 - x1)

For pair of points (1, 3) and (8, 8),

m1 = (8 - 3)/(8 - 1)

m1 = 5/7

For pair of points (8, 8)  and (15, 13),

m2 = (13 - 8)/(15 - 8)

m2 = 5/7

For pair of points (22,18)  and (29, 23),

m3 = (23 - 18)/(29 - 22)

m3 = 5/7

For pair of points (15, 13)  and (22, 18),

m4 = (18 - 13)/(22 - 15)

m4 = 5/7

Since the rate of change of output values to the input values is constant i.e., 5/7, the slope of the line is 5/7

Therefore, the slope of the line passing through given points is 5/7

Learn more about the slope here:

https://brainly.com/question/16180119

#SPJ1

(8.5-2x)(11-2x)(x) what is the approximate value of x that would allow you to construct an
open-top box with the largest volume possible from one piece of paper

Answers

The largest volume possible from one piece of paper for open-top box is 64.296 cubic unit.

What is meant by the term maxima?The maxima point on the curve will be the highest point within the given range, and the minima point will be the lowest point just on curve. Extrema is the product of maxima and minima.

For the given question dimensions of open-top box;

The volume is given by the equation;

V = (8.5-2x)(11-2x)(x)

Simplifying the equation;

V = x(4x² - 39x + 93.5)

Differentiate the equation with respect to x using the product rule.

dV/dx = x(8x -39) + (4x² - 39x + 93.5)

dV/dx = 8x² - 39x + 4x² - 39x + 93.5

dV/dx = 12x² - 72x + 93.5

Put the Derivative equals zero to get the critical point.

12x² - 72x + 93.5 = 0.

Solve using quadratic formula to get the values.

x = 4.1  and x = 1.9

Put each value of x in the volume to get the maximum volume;

V(4.1) =  4.1(4(4.1)² - 39(4.1) + 93.5)

V(4.1) = 3.44 cubic unit.

V(1.9) = 1.9(4(1.9)² - 39(1.9) + 93.5)

V(1.9) = 64.296 cubic unit. (largest volume)

Thus, the largest/maximum volume possible from one piece of paper for open-top box is 64.296 cubic unit.

To know more about the maxima, here

https://brainly.com/question/17184631

#SPJ1

An adult elephant drinks about 225 liters of water each day. Is the number ofdays the water supply lasts proportional to the numberof liters of water the elephant drinks?Is it proportional

Answers

An elephant drinks 225 liters of water per day, then in 2 days it drinks 2*225 = 450 liters, in 3 days it drinks 3*225 = 675 liters.

time (days) | 1 | 2 | 3

water (L) | 225 | 450 | 675

If an elephant drinks more water per day, the number of days the water supply lasts decrease. Then these two variables are inversely proportional.

Write a two column proofGiven: q is parallel to r Prove: angle 1 is supplementary to angle 3

Answers

Answer:

Proved.

Explanation:

Given: q is parallel to r

Statement: m∠1 = m∠2

Reason: Vertically Opposite Angles

Statement: m∠2+m∠3=180°

Reason: Same-side Interior Angles

Recall that m∠1 = m∠2

Statement: m∠1+m∠3=180°

Reason: Congruent Angles (m∠1 = m∠2)

Therefore, angle 1 is supplementary to angle 3​.

Proved.

Meri invests 15000 into an account the interest is compounded monthly for 17 years. The account balance will be 87,219.93 at the end of 17 years. What is the annual interest rate?

Answers

Annual interest rate will be 11.95% or 12% approx.

What is compound interest?

The interest earned on savings that is computed using both the original principal and the interest accrued over time is known as compound interest.

It is thought that Italy in the 17th century is where the concept of "interest on interest" or compound interest first appeared. It will accelerate the growth of a total more quickly than simple interest, which is solely calculated on the principal sum.

Money multiplies more quickly thanks to compounding, and the more compounding periods there are, the higher the compound interest will be.

P = principal

i = nominal annual interest rate in percentage terms

n = number of compounding periods

formula for compound interest is [tex]P [(1 + i)^n - 1][/tex]

According to the question

P=15000

i = ?

compound interest = 87,219.93

n=17 years

Therefore

[tex]87219.93=15000 [(1 + i)^1^7 - 1][/tex]

[tex]\frac{87219.93}{15000} = [(1 + i)^1^7 - 1][/tex]

[tex]6.814662 = (1+i)^1^7[/tex]

[tex](6.814662)^\frac{1}{17} = (1+i)[/tex]

i =1.1195-1

i =0.1195

i.e. 11.95%

Learn more about compound interest from the link below

https://brainly.com/question/14295570

#SPJ1

The rabbit population in a certain area is 200% of last year's population. There are 1100 rabbits this year. How many were there last year?

Answers

As per the given percentage, the population of rabbits in the area is 2200 in last year.

Percentage:

Percentage refers the ratio or the fraction that is multiplied and divided by 100. And it will be represented by the symbol "%".

Given,

The rabbit population in a certain area is 200% of last year's population. There are 1100 rabbits this year.

Now, we need to find the population of rabbit in last year.

Let us consider x be the total number of rabbit in last year.

We know that the rabbit in the current year is 110.

And we also know that, there are 200% of rabbits in last year.

So, we have to write it in the following expression,

200% of 1100 = x

so, the value of x is,

x = 200/100 x 1100

x = 2200

Therefore, there are 2200 rabbits in last year.

To know more about Percentage here.

https://brainly.com/question/24159063

#SPJ1

Tony used a photocopier to dilate the design for a monorail track system. The figure below shows the design and its photocopy:The ratio of CD:GH is 2:3. What is the length, in meters, of side EH on the photocopied image?

Answers

Solution

The length of EH is;

[tex]\begin{gathered} \frac{2}{3}=\frac{8}{EH} \\ \\ \Rightarrow EH=\frac{3}{2}\times8=12 \\ \\ \Rightarrow EH=12 \end{gathered}[/tex]

arina runs up 4 flights and runs down 4 flights of stairs does this situation repreasent additive inverses explain . A. Yes; The numbers combine to eight B. Yes; The numbers are combine to zero C. No;The numbers are both represented by the same integer. D. No; The numbers cannot be added together.

Answers

B. Yes; The numbers are combine to zero

is additive inverses because the sum is 0

[tex]\begin{gathered} 4+(-4) \\ =0 \end{gathered}[/tex]

A rectangular garden is 15 feet wide. If its area is 1050ft², what is the length of the garden?

Answers

The width is w=15 ft.

The area is A=1050 sq ft.

The length of the garden is,

[tex]\begin{gathered} L=\frac{A}{w} \\ =\frac{1050ft^2}{15ft} \\ =70ft \end{gathered}[/tex]

Thus, the length of the garden is 70 ft.

Find (3/5x+3/4)−(1/3x−1/8)

Answers

The answer is [tex]\frac{32x+105}{120}[/tex].

The area of mathematics known as algebra is used to represent situations or problems using mathematical expressions. In algebra, we combine integers with variables like x, y, and z.

A fraction is a number that is a component of a whole. In algebra, fractions can be added, subtracted, multiplied, and divided just like in basic arithmetic.

The given equation is an algebraic fraction having variables in the numerator. This equation is written as,

[tex]\left(\frac{3}{5}x+\frac{3}{4}\right)-\left(\frac{1}{3}x-\frac{1}{8}\right)[/tex]

First combine, x with the nearby fraction,

[tex]\begin{aligned}\left(\frac{3}{5}x+\frac{3}{4}\right)-\left(\frac{1}{3}x-\frac{1}{8}\right)&=\left(\frac{3x}{5}+\frac{3}{4}\right)-\left(\frac{x}{3}-\frac{1}{8}\right)\\&=\frac{3x}{5}+\frac{3}{4}-\frac{x}{3}+\frac{1}{8}\end{aligned}[/tex]

Now, group the fraction with a common denominator,

[tex]\begin{aligned}\left(\frac{3}{5}x+\frac{3}{4}\right)-\left(\frac{1}{3}x-\frac{1}{8}\right)&=\left(\frac{3x}{5}-\frac{x}{3}\right)+\left(\frac{3}{4}+\frac{1}{8}\right)\\&=\left(\frac{3x\cdot3}{15}-\frac{x\cdot5}{15}\right)+\left(\frac{3\cdot2}{8}+\frac{1}{8}\right)\\&=\left(\frac{9x-5x}{15}\right)+\left(\frac{6+1}{8}\right)\\&=\frac{4x}{15}+\frac{7}{8}\\&=\frac{32x+105}{120}\end{aligned}[/tex]

Therefore, the final answer is [tex]\frac{32x+105}{120}[/tex].

To know more about algebraic fractions:

https://brainly.com/question/10354322

Emily reads 22 pages per hour. In all, how many hours of reading will Emily have to do this week in order to have read a total of 44 pages?

Answers

Answer:

The number of hours of reading it will take Emily to read a total of 44 pages is;

[tex]2\text{ hours}[/tex]

Explanation:

Given that Emily reads 22 pages per hour.

Her rate of reading is;

[tex]r=22\text{ pages/hour}[/tex]

The amount of time she needs to read 44 pages will be the number of pages divided by the rate;

[tex]\begin{gathered} t=\frac{\text{number of pages}}{\text{rate}}=\frac{44}{22}\text{hour} \\ t=2\text{ hours} \end{gathered}[/tex]

Therefore, the number of hours of reading it will take Emily to read a total of 44 pages is;

[tex]2\text{ hours}[/tex]

2 hours because 22 plus 22 is 44, and you only add 22 2 times

When asked to find f ( g ( x ) ), what should you do? plug f (x) into g (x)plug g (x) into f (x)multiply f (x) by g (x)none of the above

Answers

From the given question,

f(g(x)) means,

Plug g(x) into f(x)

Hence, the correct option is B

A) Find the simple interest amount earned for $5500 at 6.5% for 5 months. b)What is the total value of the investment?

Answers

The simple interest I on an amount P invested at an interest rate R %, for a period of time T per annum is evaluated as

[tex]I\text{ = }P\times R\times T[/tex]

A) The interest earned at $5500 at 6.5% for 5 months is thus evaluated as

[tex]\begin{gathered} P\text{ = 5500} \\ R\text{ = 6.5\% = }\frac{6.5}{100} \\ T\text{ = 5 months = }\frac{5}{12}\text{ year} \\ thus, \\ I\text{ = }5500\times\frac{6.5}{100}\times\frac{5}{12} \\ \Rightarrow I\text{ = \$ 148.958} \end{gathered}[/tex]

thus, the interest earned for $5500 at 6.5% for 5 months is $ 148.985.

B) Total value of the investment.

The total value of the investment is the sum of the interest earned and the initial amount invested.

Thus,

[tex]\begin{gathered} Total\text{ value of investment = interest earned + amount invested} \\ A\text{ = I + P} \\ A\text{ = 148.985 + 5500} \\ \Rightarrow A\text{ = \$ 5648.985} \end{gathered}[/tex]

Hence, the total value of the investment is $ 5648.985.

Hi, can you help me to find (Ir possible) the complement andsupplement of the angle of exercise

Answers

The angle is given 24 degree.

To determine the complement angle ,

[tex]90^{\circ}-24^{\circ}=66^{\circ}[/tex]

To determine the supplement angle ,

[tex]180^{\circ}-24^{\circ}=156^{\circ}[/tex]

Find FG.FL x + 113x + 1EН.FG=

Answers

Answer:

FG = 16

Explanation:

The triangles EFG and EHG are congruent because they share the side EG and they have the same interior angles.

If they are congruent the lengths of the corresponding sides are equal, so we can write the following equation:

FG = GH

Then, subtitute FG = x + 11 and GH = 3x + 1 to get:

x + 11 = 3x + 1

So, solving for x, we get:

x + 11 = 3x + 1

x + 11 - 1 = 3x + 1 - 1

x + 10 = 3x

x + 10 - x = 3x - x

10 = 2x

10/2 = 2x/2

5 = x

Then, replacing x by 5, we get that FG is equal to:

FG = x + 11

FG = 5 + 11

FG = 16

So, the answer is FG = 16

Find all possible rational roots of f(x)=4x^3-13x^2+9x+2

Answers

Polynomial

[tex]f(x)=4x^3+13x^2+9x+2[/tex]

I need help with this, I need it step by step please.

Answers

In this problem we have a translation

the rule is

(x,y) ------> (x-6,y)

that means

the translation of the point is 6 units at left

so

we have

E(2,4) ------> E'(2-6,4)

E'(-4,4)

you must to subtract 6 units from the x coordinate

F(4,4) -----> F'(4-6,4)

F'(-2,4)

G(2,1) -----> G'(2-6,1)

G'(-4,1)

In 10 seconds, Jake travels 550 feet on his bike. At this speed. How many fert can he travel in 1 minute.

Answers

We can find out how many feet can Jake travel using a rule of three:

[tex]\begin{gathered} 10s\rightarrow550ft \\ 60s\rightarrow xft \\ \Rightarrow x=\frac{60\cdot550}{10}=3300 \\ x=3300ft \end{gathered}[/tex]

therefore, Jake can travel 3300ft in 1 minute

Solve the system by the method of your choice. Identify inconsistent systems and systems with dependent equations, using set notation to express solution sets

Answers

The given system of equations is

[tex]\begin{gathered} y=3x+5\rightarrow(1) \\ 5x-2y=-7\rightarrow(2) \end{gathered}[/tex]

Substitute y in equation (2) by equation (1)

[tex]5x-2(3x+5)=-7[/tex]

Simplify the left side

[tex]\begin{gathered} 5x-2(3x)-2(5)=-7 \\ 5x-6x-10=-7 \end{gathered}[/tex]

Add the like terms on the left side

[tex]\begin{gathered} (5x-6x)-10=-7 \\ -x-10=-7 \end{gathered}[/tex]

Add 10 to both sides

[tex]\begin{gathered} -x-10+10=-7+10 \\ -x=3 \end{gathered}[/tex]

Divide both sides by -1

[tex]\begin{gathered} \frac{-x}{-1}=\frac{3}{-1} \\ x=-3 \end{gathered}[/tex]

Substitute x in equation (1) by -3 to find y

[tex]\begin{gathered} y=3(-3)+5 \\ y=-9+5 \\ y=-4 \end{gathered}[/tex]

The solution of the system of equations is {(-3, -4)}

Since the system has only one solution then it is an independent consistent system.

Debra is going to rent a truck for one day. There are two companies she can choose from, and they have the following prices: company A has no initial fee but charges 80 cents for every mile driven. Company B charges an initial fee of $75 and an additional 70 cents for every mile driven. For what mileages will company A charge at least as much as company B? Use m for the number of miles driven, and solve your inequality for m.

Answers

Given that m is the number of miles driven by the truck.

From the information given,

The charge by Company A would be:

[tex]0+80\times m=80m[/tex]

The charge by Company B would be:

[tex]75+70\times m=75+70m[/tex]

If company A charges at least as much as company B, then

[tex]A\ge B[/tex]

Therefore,

[tex]\begin{gathered} 80m\ge75+70m \\ 80m-70m\ge75 \\ 10m\ge75 \\ m\ge\frac{75}{10}=7.5 \\ \text{The mileage is 7.5}\frac{miles}{cent} \end{gathered}[/tex]

Find the area of the figure. (Sides meet at right angles.) Check 7 yd 5 yd 3 yd 3 yd 13 yd 3 yd 5 yd 7 yd yd²

Answers

Given:

The figure with sides measurements.

Required:

Find area of the figure.

Explanation:

First we will draw figure

In figure, we can see that all figure ABIJ, CDHI and EFGH are rectangles.

So, we need area of rectangle formula. That is

[tex]A=length\times width[/tex]

So, area of given figure

[tex]\begin{gathered} A=\text{ Area of ABIJ +Are of CDHI + Area of EFGH} \\ ABIJ=EFGH \\ So, \\ A=2\times(ABIJ)+CDHI \\ A=2\times(7\times5)+(4\times3) \\ A=70+12 \\ A=82yd^2 \end{gathered}[/tex]

So the question is A local theater sells admission tickets for $9.00 on Thursday nights, where n is the number of customers M(n) the amount of money the theater takes What is the domain of M( n ) in this context

Answers

Domain of a function

We have the function M(n):

M( n ) = 9 · n

which describes the amount of money the theater takes.

Since the domain of a function refers to the values n can take and n is the number of costumers

In this particular case we do not have a restriction of the number of costumers. Then n can take the following values:

n = 0, 1 , 2, 3, ...

Domain: all non- negative integers

If the maximum capacity of the theater is 100 costumers then

n = 0, 1, 2, 3, ..., 100

Therefore its domain would correspond to

Domain: all non- negative integers less than or equal to 100

Consider the following expression and determine which statements are true. z? + 5y: -8 Choose 2 answers: There are 3 terms. The variables are z, y. and . The coefficient of zis 2 The term Syz is made up of 2 factors.

Answers

we have:

the expression has 3 terms and 3 variables, they are x, y and z. Therefore

answer:

A and B

*Express the end behavior of the followingFunction in limit notation.G(x)=-x(x^2 + 3) (x - 2)^3 (x + 5)^2

Answers

we have the function

[tex]g(x)=-x(x^2+3)(x-2)^3(x+5)^2[/tex]

In this problem, we have that

the leading coefficient is negative (-1)

The degree of the function is 8 (even)

therefore

the end behavior of the function is

f(x)→−∞, as x→−∞

f(x)→−∞, as x→+∞

Find x, where a=14 degrees and b=22 degrees. Find the measure of each angle of the polygon. Shown below.

Answers

To answer this question, we need to remember that the sum of the interior angles of a quadrilateral is equal to 360º (in fact, we can divide a quadrilateral into two triangles, and the sum of interior angles of a triangle is equal to 180º).

Then, we have that a = 14º and b = 22º, then we can state the next equation:

[tex](2x+14^{\circ})+(3x+22^{\circ})+2x+x=360^{\circ}[/tex]

And now, we can solve the equation for x as follows:

1. Add the like terms as follows:

[tex](2x+3x+2x+x)+14^{\circ}+22^{\circ}=360^{\circ}[/tex][tex]8x+36^{\circ}=360^{\circ}[/tex]

2. Subtract 36º from both sides of the equation:

[tex]8x+36^{\circ}-36^{\circ}=360^{\circ}-36^{\circ}\Rightarrow8x=324^{\circ}[/tex]

3. Divide both sides by 8 as follows:

[tex]\frac{8x}{8}=\frac{324^{\circ}}{8}\Rightarrow x=40.5^{\circ}[/tex]

Therefore, the value for x = 40.5º

Then, we can find the values for the measure of angle A as follows:

[tex]m\angle A=2(40.5^{\circ})+14^{\circ}=95^{\circ}[/tex]

The measure of angle B is

[tex]m\angle B=3(40.5^{\circ}_{})+22^{\circ}=143.5^{\circ}[/tex]

The measure of angle C is

[tex]m\angle C=x^{\circ}=40.5^{\circ}[/tex]

The measure of angle D is

[tex]m\angle D=2(40.5^{\circ})=81^{\circ}[/tex]

Donovan took a math test and got 20 correct questions and 5 incorrect answers. What was the percentage of correct answers?

Answers

ANSWER

80%

EXPLANATION

Donovan got 20 questions correct and 5 questions incorrect.

This means that the total number of questions he attempted in the test is the sum of correct and incorrect questions:

Total = 20 + 5

Total = 25

To find the percentage of correct answers, we have to divide the number of correct answers by the total number of questions attempted and multiply by 100 (per cent).

That is:

[tex]\begin{gathered} \frac{20}{25}\cdot\text{ 100 = }\frac{4}{5}\cdot\text{ 100} \\ =\text{ 80\%} \end{gathered}[/tex]

That is the percentage of correct answers.

Other Questions
Write down the expansion of (2x+y)^4 why does omnibus legislation require alternatives to the committee system? find the perimeter and area It is Nadines birthday, and she is making walnut brownies for her sleepover. She invited eight of her friends to her party. The recipe that she is using makes twenty-four brownies, but she only wants to make eight. The recipe for twenty-four calls for \large 4\frac{1}{2} cups of walnuts. How many cups of walnuts will she need for eight people? hXL for School: Practice & Problem Solving 5.2.PS-19 Question Help Equivalent ratios can be found by extending pairs of rows or columns in a multiplication table. Write 3 3 ratios equivalent to using the multiplication table. 5 Click the icon to view the multiplication table. 3 Find three ratios that are equivalent to 5 12 6 4 IA. B. OC. 20 10 6 15 15 9 OD OE. F. 9 30 15 Click to select your answer(s) and then click Check Answer. All parts showing Clear All Check Answer Review progress Question 7 of 12 Back Next > A food truck caters an event attended by 100 guests. Every guest orders one of two possible dishes: a salad or a turkey plate. The price of each meal decreases as more of that particular type are ordered. The price of a salad is $10.00 minus $0.06 for each salad ordered. The price of a turkey plate is $12.00 minus $0.02multiplied by the square of the number of turkey plates ordered. Guests pay for their meal only after everyone has placed their order.Using differentiation, find the maximum revenue for the food truck.Remember that the number of meals is a positive integer. Round revenue to the nearest cent. Based on the audio, what may the villagers do after the natural disaster? Graph the linear function g(x) = -4+7xGraph the linear function G (x equals negative 4 + 7x If there are three black, four white, two blue, and four gray socks in a drawer, what would be the probability of picking a blue sock? Round your answer to the nearest tenth.15.4%25%22%15.38% In the image below LMOP. Given the lines are parallel, LMNPON because and that LNMONP by the , you can conclude the triangles are similar by the AA Similarity Theorem. If NP = 20, MN = x+ 6, NO = 15, and LN = 2x - 3 then x = . 76) In bacteria, the unit of DNA that contains multiple genes under the control of a single promoter is called ____ this unit is referred to as _____ mRNAa. an operator, a polycistronicd. an operon, a monocistronicb. a template, a structurale. a template, a monocistronicc. an operon, a polycistronic z divided by 13=28 i need the answers this is hard for me The graph shows a relationship between two quantities.200018001600140012001000800600400200Od-8 -6 4-2 0 2 4Which equation best represents the relationship between the variables? Find the reference angle for a rotation of 129. are there two types of addiction, psychologicaland physical. How do you figure out what the order pairs are in this equation? 2x-2=y Determine the smallest integer value of x in the solution of the following inequality.3x + 4> -18 Every rational number is also an integer.TrueorFalse Put the following equation of a line into slope-intercept form, simplifying allfractions.12y-2x = 108 a cannon sitting on top of a 40 m mound shoots a projectile with an initial velocity of 4.47 m/s parallel to ground. how far away did it land?